Compare the functions shown below: f(x) = (x + 3)2 − 2 g(x) linear graph with y intercept of negative 3 over 2 and x intercept of 3 h(x) x y −3 2 −2 7 −1 14 0 23 1 34 2 47 3 62 What is the correct order of the functions from least to greatest according to the average rate of change on the interval from x = −1 to x = 3?

Answers

Answer 1

Answer:

For f (x):

f (x) = (x + 3) ^ 2 - 2

For x = -1

f (-1) = (-1 + 3) ^ 2 - 2

f (-1) = (2) ^ 2 - 2

f (-1) = 4 - 2

f (-1) = 2

For x = 3

f (3) = (3 + 3) ^ 2 - 2

f (3) = (6) ^ 2 - 2

f (3) = 36 - 2

f (3) = 34

AVR = ((34) - (2)) / ((3) - (- 1))

AVR = 8

For g (x):

linear graph with and intercept of negative 3 over 2 and x intercept of 3

y = mx + b

b = -3/2

For me we have:

0 = m (3) - 3/2

3m = 3/2

m = 1/2

The function g (x) is:

g (x) = (1/2) x - 3/2

For x = -1

g (-1) = (1/2) (- 1) - 3/2

g (-1) = -1/2 - 3/2

g (-1) = -4/2

g (-1) = -2

For x = 3

g (3) = (1/2) (3) - 3/2

g (3) = 3/2 - 3/2

g (3) = 0

AVR = ((0) - (- 2)) / ((3) - (- 1))

AVR = 1/2

For h (x):

Using the table we have:

AVR = ((62) - (14)) / ((3) - (- 1))

AVR = 12

Step-by-step explanation:

Answer 2

Answer:

from least to greatest:

1) g (x)

2) f (x)

3) h (x)

Step-by-step explanation:


Related Questions

Please answers question 5 (the line under H is labeled E, just got cut off the picture

Please answers question 5 (the line under H is labeled E, just got cut off the picture

Answers

Answer:

150

Step-by-step explanation:

Knowledge

hello can you help me with this question? 15 points

hello can you help me with this question? 15 points

Answers

Hi! This problem has a lot to do with “like-terms” also known as terms (similarly formatted numbers/variables).

1st box:
6x and 8x are being combined.
6x-8x=-2x

2nd box:
-4x and -2x (from the 1st box) are being combined.
-2x+4x=2x

3rd box:
6 and 2 are being combined.
6-2=4

Altogether, we would have 4=2x. With one last division of 4/2, we would get 2=x.

Don’t be afraid to ask if you have any more questions! :))

It cost Liam $6.10 to send 61 text messages. How much would it cost to send 185 text messages?

Answers

Answer:

$18.50

Step-by-step explanation:

In the example before 61 messages = $6.10 or rounding to the tenths place 6.1. If you do the same with 185, you get 18.5 or 18.50

Answer:

$18.50

Step-by-step explanation:

6.10 times 10 = 61

185 divided by 10 = 18.5

Hannah lines up 6 plastic letters on a sign t spell her name. Each letter takes up 5/8 inch of space on the sign.
The total amount of space taken up on the sign by the letters in Hannah's name is between which two whole numbers?

Answers

The total amount of space taken up on the sign by the letters in Hannah's name is between 3 and 4 whole numbers.

According to given condition :

If each letter takes up 5/8 inch of space, then the total amount of space taken up by all 6 letters in Hannah's name is:

6 letters × 5/8 inch per letter = 30/8 inches = 3 6/8 inches = 3 3/4 inches

To find the range of whole numbers between which 3 3/4 inches falls, we can convert 3 3/4 to an improper fraction:

3 3/4 = (4 × 3 + 3)/4 = 15/4

Now we can find the two whole numbers that 15/4 is between by dividing 15 by 4:

15 ÷ 4 = 3 with a remainder of 3

So 15/4 is between 3 and 4, specifically:

3 < 15/4 < 4

Therefore, the total amount of space taken up on the sign by the letters in Hannah's name is between 3 and 4 whole numbers.

What is fraction ?

A fraction is a mathematical concept that represents a part of a whole. It is typically written in the form of one number (the numerator) over another number (the denominator), separated by a horizontal line. For example, 3/4 is a fraction where the numerator is 3 and the denominator is 4.

The numerator represents how many parts of the whole we are referring to, and the denominator represents the total number of equal parts that make up the whole. So, 3/4 represents 3 out of 4 equal parts of a whole. Fractions are used to represent quantities that are not whole numbers, such as 1/2, 3/5, and 7/8.

Fractions are used in many different areas of math and everyday life, such as in measurement, cooking, and finance. They are also an important concept in understanding other mathematical ideas like decimals and percentages.

To know more about fraction visit :

https://brainly.com/question/78672

#SPJ1

Please help! Tysm if you do!

Please help! Tysm if you do!

Answers

Answer:

A

Step-by-step explanation:

We can solve this problem by eliminating wrong answers

It can be b because there is no 3

It can be c because the equation would be 4+3

And it can be d because the equation would be 3x+4

so it must be a

Answer:

option A is correct

as additional page costs is not fixed so there will be any value of x with multiply by 4

and the fixed remain only one time so it remains fixed and 3 is also fixed.

What is the area of a circle with diameter = 4ft, using 3.14 for ? thx :)

Answers

Answer:

12.56

Step-by-step explanation:

I'm 99% sure the answer is 12.57ft²

Are these answers correct or no?

Are these answers correct or no?

Answers

Answer:

Yes, they are correct.

Step-by-step explanation:

\( \frac{4}{10} = \frac{2}{5} \)

\( \frac{6}{15} = \frac{2}{5} \)

Hope it helps :)

If angle 5=91-2x and angle 10=5x find the value of x

Answers

The value of x for the same angle based on the information is 13.

How to calculate tie angle

In Plane Geometry, a figure which is formed by two rays or lines that shares a common endpoint is called an angle

Since angles 5 and 10 are the same, we can set them equal to each other:

91 - 2x = 5x

Simplifying this equation, we can add 2x to both sides:

91 = 7x

Then, we can divide both sides by 7 to solve for x:

x = 13

Therefore, the value of x for the same angle is 13.

Learn more about angle on

https://brainly.com/question/25716982

#SPJ1

Write an expression that matches the difference between 8 forty-sevens and 5 forty-sevens.

Answers

(8 x 47) - (5 x 47)

Also The difference is 141

\(y \geq \frac{1}{3}x+4\\\)

Answers

In linear equation, y ≥ x + 12/3 is simple version .

What in mathematics is a linear equation?

There are only one or two variables in a linear equation. No variable can be multiplied by a number larger than one or used as the denominator of a fraction in a linear equation. All of the points fall on the same line when you identify the values that together make a linear equation true and plot those values on a coordinate grid.It has the formula Ax + B = 0, with A and B being any two real numbers and x being an ambiguous variable with only one possible value. One such linear equation in one variable is 9x + 78 = 18.

y ≥ 1/3 x + 4

y ≥ x/3 + 4

y ≥ x + 12/3

Learn more about linear equation

brainly.com/question/11897796

#SPJ9

Use the multiplication rule to simplify:
67.6²

Answers

4569.76 you cannot simplify that

I'm guessing that you want to simplify \(67.6^2\)

The two above the 67.2 is a square or a power.

Usually when you take something to a power, you mutiply the number of time by the power:

Examples:   \(78^2\) would be 78*78

                   \(23^5\) would be 23*23*23*23*23

Questions?

67.6² would simply be 67.6*67.6.

So therefore it is 4569.76

Students were asked how they travel to school. The graph shows the results of the survey. Based on the graph, how many students out of 300 students can be expected to walk or bike to school?

Answers

Answer:

we need the graph in order to find the answer.

Step-by-step explanation:

The square of a number is equal to two less than three times the number. What are two possible values of the number?
A. 1,2
B. -1,2
C. 1,-2
D. -1,-2
E. 2,3

Answers

Answer:

A, 1,2

Step-by-step explanation:

we translate it first into language of math

we use n for the number

n² = 3n - 2

then, do some algebra things

n² - 3n + 2 = 0

then factorize it

(n - 2)(n - 1) = 0

first n

n - 2 = 0

n = 2

2nd n

n - 1 = 0

n = 1

so n can be 1 or 2

Give simple working out

Give simple working out

Answers

The missing angle of the diagram is expressed as; x = 70°

How to find the sum of angles at a point?

An angle is usually measured with reference to a circle with its center at the common endpoint of the rays. Hence, the sum of angles at a point is always 360 degrees

Now, we are given a point with three angles where one is unknown and the other two are known. Thus, by the sum of angles about a point, they will sum up to 360 degrees.

Thus;

x + 160 + 130 = 360

x + 290 = 360

x = 360 - 290

x = 70°

Read more about sum of angles at a point at; https://brainly.com/question/25055291

#SPJ1

Let v1 = 1 , v2 = 4 , v3 = 11 , and w = 50 1 3 1-1 5 16 4a. Is w in v1, V2, V3]? How many vectors are in fv1, V2. v3]? b. How many vectors are in Span v1, V2. V3)? c. Is w in the subspace spanned by {vj, V2, V3]? Why?

Answers

a. w is not in {v1, v2, v3}. b. There are infinitely many vectors in Span{v1, v2, v3}. c. No, w is not in the subspace spanned by {v1, v2, v3} because it is not a linear combination of these three vectors.

a. To determine if w is in {v₁, v₂, v₃}, we need to check if w is equal to a linear combination of v₁, v₂, and v₃. That is, we need to check if there exist constants c1, c₂, and c₃ such that:

c₁v₁ + c₂v₂ + c₃v₃ = w

Substituting the given values, we get:

c₁(1) + c₂(4) + c₃(11) = 50

Solving this system of equations, we find that there are no values of c₁, c₂, and c₃ that satisfy the equation. Therefore, w is not in {v₁, v₂, v₃}.

b. The set {v₁, v₂, v₃} has three vectors, and any linear combination of these vectors will be in the span of {v₁, v₂, v₃}. Therefore, to find the number of vectors in Span {v₁, v₂, v₃}, we need to determine how many linear combinations of these vectors are possible. Each vector in the span can be written as a linear combination of v₁, v₂, and v₃, so there are infinitely many vectors in Span {v₁, v₂, v₃}.

c. Since w is not in {v₁, v₂, v₃}, we need to check if w can be written as a linear combination of {v₁, v₂, v₃}. If w is in the subspace spanned by {v₁, v₂, v3}, then there exist constants c₁, c₂, and c₃ such that:

c₁v₁ + c₂v₂ + c₃v₃ = w

As we found in part (a), there are no values of c₁, c₂, and c₃ that satisfy this equation. Therefore, w is not in the subspace spanned by {v₁, v₂, v₃}.

Learn more about vectors here: brainly.com/question/29740341

#SPJ4

number story for 4 divided by 1/8

Answers

Answer:

32

Step-by-step explanation:

4 ÷ 1/8

4/1 · 8/1 = 32/1

32

I hope this helps & God bless!

The most common purpose for Pearson correlational is to examine

Answers

For Pearson correlation the most common purpose to examine is given by option a. The relationship between 2 variables.

The Pearson correlation is a statistical measure that indicates the extent to which two continuous variables are linearly related.

It measures the strength and direction of the relationship between two variables.

Ranging from -1 perfect negative correlation to 1 perfect positive correlation.

And with 0 indicating no correlation.

It is commonly used in research to examine the association between two variables.

Such as the relationship between height and weight, or between income and education level.

Therefore, the most common purpose of a Pearson correlation is to examine the relationship between 2 variables.

learn more about correlation here

brainly.com/question/30433001

#SPJ4

The above question is incomplete, the complete question is:

The most common purpose for a Pearson correlation is to examine,

a. The relationship between 2 variables

b. Relationships among groups

c. Differences between variables

d. Differences between two or more groups

. True or false. Randomization distributions are centered at the value where the null is true. True False

Answers

False. Randomization distributions are centered at the value where the null is false. This distinction is important in hypothesis testing as it helps determine the statistical significance of observed differences or effects.

Randomization distributions are generated through permutation tests, which involve random shuffling of data points between groups. In hypothesis testing, the null hypothesis assumes that there is no difference between groups or no effect of a treatment. The alternative hypothesis suggests otherwise.

The randomization distribution is created by repeatedly shuffling the data and calculating a test statistic based on the null hypothesis. This distribution represents the expected outcomes if the null hypothesis is true. Therefore, the randomization distributions are centered at the value where the null is false, as they capture the potential differences or effects between groups.

In summary, randomization distributions are not centered at the value where the null is true; rather, they are centered at the value where the null is false. This distinction is important in hypothesis testing as it helps determine the statistical significance of observed differences or effects.

To know more about hypothesis visit:

https://brainly.com/question/32562440

#SPJ11

Solve the system by Substitution.
x- 4y=-7
-x +2y = 3

Answers

Intro:

Hello!!! Princess Sakura here ^^

Step-by-step explanation:

Step 1:

Solve x−4y=−7 for x...

\(x-4y=-7\\x=4y-7\)

Step 2:

Substitute 4y−7 for x in −x+2y=3...

\(-x+2y=3\\-(4y-7)+2y=3\\-4y+7+2y=3\\-2y+7=3\\-2y=-4\\y=2\)

Step 3:

Substitute 2 for y in x=4y−7...

\(x=4y-7\\x=4(2)-7\\x=8-7\\x=1\)

So the solution of these equations is (1,2).

In a recent Game Show Network survey, 30% of 5000 viewers are under 30. What is the margin of error at the 99% confidence interval? Using statistical terminology and a complete sentence, what does this mean? (Use z*=2. 576)



Margin of error:



Interpretation:

Answers

The margin of error at the 99% confidence interval is 0.018 or 1.8%.

Interpretation: This means that if we were to repeat the survey many times, about 99% of the intervals calculated from the samples would contain the true proportion of viewers under 30 in the population, and the margin of error for each interval would be no more than 1.8%.

The margin of error is the amount by which the sample statistic (in this case, the proportion of viewers under 30) may differ from the true population parameter.

Using the given formula for margin of error:

Margin of error = z* * sqrt(p*(1-p)/n)

Where:

- z* is the z-score corresponding to the confidence level (99% in this case), which is 2.576

- p is the proportion of viewers under 30, which is 0.3

- n is the sample size, which is 5000

Substituting these values, we get:

Margin of error = 2.576 * sqrt(0.3*(1-0.3)/5000) = 0.018

To know more about margin of error refer to

https://brainly.com/question/10218601

#SPJ11

suppose that the distribution for total amounts spent by students vacationing for a week in florida is normally distributed with a mean of 650 and a standard deviation of 120 . suppose you take a simple random sample (srs) of 20 students from this distribution. what is the probability that a srs of 20 students will spend an average of between 600 and 700 dollars? round to five decimal places.

Answers

The probability that a srs of 20 students will spend an average of between 600 and 700 dollars is 0.92081.

We need to find the probability that a simple random sample of 20 students will spend an average of between 600 and 700 dollars.

To solve this problem, we will use the central limit theorem, which states that the sampling distribution of the sample means will be approximately normally distributed with a mean of μ and a standard deviation of σ/√(n), where n is the sample size.

Thus, the mean of the sampling distribution is μ = 650 and the standard deviation is σ/sqrt(n) = 120/√(20) = 26.83.

We need to find the probability that the sample mean falls between 600 and 700 dollars. Let x be the sample mean. Then:

Z1 = (600 - μ) / (σ / √(n)) = (600 - 650) / (120 / √t(20)) = -1.77

Z2 = (700 - μ) / (σ / √(n)) = (700 - 650) / (120 / √(20)) = 1.77

Using a standard normal distribution table or calculator, we can find the area under the standard normal distribution curve between these two Z-scores as:

P(-1.77 < Z < 1.77) = 0.9208

Therefore, the probability that a simple random sample of 20 students will spend an average of between 600 and 700 dollars is 0.9208, or approximately 0.92081 when rounded to five decimal places.

Learn more about probability here:

brainly.com/question/14228383

#SPJ11

Each day that Barney goes to college, he either goes by bus or he walks.
The probability that Barney will go to college by bus on any day is 0.3

When Barney goes to college by bus, the probability that he will be late is 0.2
When Barney walks to college, the probability that he will be late is 0.1

Barney will go to college on 200 days next year.

Work out an estimate for the number of days Barney will be late for college next year.

Answers

To estimate the number of days Barney will be late for college next year, we need to consider the probabilities of him going by bus or walking, as well as the probabilities of him being late in each case.

Let's break down the problem step by step:

1. Probability of going by bus: The probability that Barney will go to college by bus on any given day is 0.3. Since he will go to college on 200 days next year, we can estimate that he will go by bus on 0.3 * 200 = 60 days.

2. Probability of being late when going by bus: The probability that Barney will be late when he goes to college by bus is 0.2. Therefore, on the 60 days he goes by bus, we can estimate that he will be late on 0.2 * 60 = 12 days.

3. Probability of walking: Since Barney either goes by bus or walks, the remaining days (200 - 60 = 140) are the days he walks to college.

4. Probability of being late when walking: The probability that Barney will be late when he walks to college is 0.1. Therefore, on the 140 days he walks, we can estimate that he will be late on 0.1 * 140 = 14 days.

Finally, to estimate the total number of days Barney will be late for college next year, we sum the estimated days for each mode of transportation: 12 days (going by bus) + 14 days (walking) = 26 days.

Therefore, an estimate for the number of days Barney will be late for college next year is 26 days.

How do you prove the third angle theorem?

Answers

Prove the third angle theorem is the sum of the angles in each triangle is 180 degree by the Triangle Sum Theorem

What is Third angle theorem?

A triangle's outside angle has a measure greater than either of the distant interior angles. Because the parallel postulate is not required for its proof, this is a fundamental conclusion in absolute geometry.

Concept:

You're undoubtedly rather familiar with triangles by this point. You've been using triangular building blocks since you were in preschool, and you already know a lot about their length, area, and perimeter. What about those annoying angles, though? Would it really matter if they were all triangles, say 45-45-90 or 30-60-90? At least then you'd be able to recall everything you've ever needed to deal with them.

According to the third angle theorem, the third set of angles must likewise be congruent if two angles in one triangle are congruent with two angles in another triangle. What more details would you need to be aware of in order to establish that the triangles are congruent?

You require some knowledge of the side information for the triangles to be congruent. The triangles will be congruent if you are aware of two pairs of congruent angles and at least one pair of corresponding sides.

Using the Triangle Sum Theorem, demonstrate that the third angle theorem states that the sum of the angles in each triangle is 180 degrees.

To learn more about Angle visit:

brainly.com/question/28451077

#SPJ4

A streaming service has a new movie downloaded 217 times each minute.
How many downloads are there in one day?
downloads

Answers

Answer:

312,480 downloads in one day

Step-by-step explanation:

There are different ways to approach this problem, but one possible method is to use unit conversions and basic arithmetic operations.

Here's how:

First, we need to know how many minutes are in a day. Since there are 24 hours in a day and 60 minutes in an hour, we can multiply those numbers:

24 x 60 = 1440 minutes

Next, we can use the given rate of downloads per minute (217) and multiply it by the total number of minutes in a day:

217 x 1440 = 312,480

Therefore, there are 312,480 downloads in one day.

what is the principal reason why you should use anova instead of several t tests to evaluate mean differences when an experiment consists of three or more treatment conditions? anova is better suited to expressing the treatment condition as a discrete variable. multiple t tests are prohibitively expensive and time consuming. multiple t tests accumulate the risk of a data entry error. anova, as a more advanced technique, makes a research report appear more professional. multiple t tests accumulate the risk of a type i error.

Answers

ANOVA is a more appropriate and efficient method for evaluating mean differences when an experiment consists of three or more treatment conditions, as it provides a more robust and reliable analysis than multiple t-tests, while also reducing the risk of Type I errors.

The principal reason why you should use ANOVA (Analysis of Variance) instead of several t-tests to evaluate mean differences when an experiment consists of three or more treatment conditions is that multiple t-tests accumulate the risk of a Type I error.

A Type I error is the rejection of a true null hypothesis, which means that you have falsely concluded that there is a significant difference between the treatment groups when there is actually no difference. When conducting multiple t-tests, the probability of making a Type I error increases with the number of tests performed. In other words, if you perform multiple t-tests, the overall probability of making at least one Type I error increases, and this can lead to incorrect conclusions.

ANOVA is a statistical technique that allows you to compare the means of three or more treatment groups simultaneously while controlling the probability of making a Type I error. ANOVA compares the variance between groups to the variance within groups to determine if the differences between the treatment groups are statistically significant. By using ANOVA instead of multiple t-tests, you can reduce the overall risk of making a Type I error while still evaluating the mean differences between the treatment groups.

Therefore, ANOVA is a more appropriate and efficient method for evaluating mean differences when an experiment consists of three or more treatment conditions, as it provides a more robust and reliable analysis than multiple t-tests, while also reducing the risk of Type I errors.

To learn more about ANOVA

https://brainly.com/question/31795618

#SPJ11

estimate the error that is made by approximating the sum of the given series by the series the fitst 5 terms 1/k^3

Answers

The error involved in approximating the sum of the given series by the sum of the first five terms of the series is `R5 = 1/6³ + 1/7³ + ...`.

To estimate the error that is made by approximating the sum of the given series by the series the first 5 terms 1/k³, we can use the remainder term of a convergent series.

The given series is ∑ 1/k³ from k = 1 to infinity.We have to find the error involved in approximating the sum of the given series by the sum of the first five terms of the series.

That is, we need to find the difference between the actual sum of the series and the sum of the first five terms of the series.

The sum of the series is given by: `S = 1/1³ + 1/2³ + 1/3³ + 1/4³ + ... + 1/n³ + ...` We can use the remainder term of the series to find the error in approximation.

The remainder term `Rn` is given by: `Rn = Sn - S` where `Sn` is the sum of the first `n` terms of the series. Thus, we have to find the remainder term for `n = 5`.

The remainder term `Rn` is given by: `Rn = S - Sn = 1/6³ + 1/7³ + ...` Since the given series is convergent, the remainder term `Rn` tends to zero as `n` tends to infinity.

So, if we take the sum of the first five terms of the series, the error involved in approximation is given by the remainder term `R5`.

The error involved in this approximation is very small and can be neglected.

To learn more about : terms

https://brainly.com/question/30643700

#SPJ8

At the city museum, child admission is $6.20 and adult admission is $9.50. On Friday, three times as many adult tickets as child tickets were sold, for a total sales of $1179.80. How many child tickets were sold that day? Number of chad tickets:

Answers

Answer:

34 child tickets

Step-by-step explanation:

let c = # child tickets

let a = # adult tickets

System of equations:

6.20c+9.50a=1179.80

a=3c

substitute 3c for a:

6.20c +9.50(3c) = 1179.80

34.7c=1179.80

c=34

Answer:

34 child tickets were sold that day.

Step-by-step explanation:

Let's use C to represent the number of child tickets sold and A to represent the number of adult tickets sold.

From the problem, we know that:

Child admission = $6.20Adult admission = $9.50A = 3C (since three times as many adult tickets as child tickets were sold)Total sales = $1179.80

We can set up an equation based on the information given:

\(\qquad\large\rm{6.2C + 9.5A = 1179.8}\)

Substitute A with 3C:

\(\qquad\large\rm{6.2C + 9.5(3C) = 1179.8}\)

Simplify:

\(\qquad\large\rm{6.2C + 28.5C = 1179.8}\)

\(\qquad\qquad\large\rm{34.7C = 1179.8}\)

\(\qquad\qquad\quad\large\boxed{\boxed{\bold{\:\:C \approx 34\:\:}}}\)

\(\therefore\) 34 child tickets were sold that day.

Solve the following system.

y = -2x^2 + 3x - 1 and x + y = -1.

The solutions are
and

Answers

Given:

The equations of a system of equations are

\(y=-2x^2+3x-1\)

\(x+y=-1\)

To find:

The solution of the given system of equations.

Solution:

We have,

\(y=-2x^2+3x-1\)           ...(i)

\(x+y=-1\)                      ...(ii)

Using (i) and (ii), we get

\(x+(-2x^2+3x-1)=-1\)

\(x-2x^2+3x-1+1=0\)

\(-2x^2+4x=0\)

Taking out the common factors.

\(2x(-x+2)=0\)

Using zero product property,we get

\(2x=0\) and \(-x+2=0\)

\(x=0\) and \(2=x\)

For x=0, we have

\(0+y=-1\)

\(y=-1\)

For x=2, we have

\(2+y=-1\)

\(y=-1-2\)

\(y=-3\)

Therefore, the two solutions of the given system of equations are (0,-1) and (2,-3).

help me find x please ​

help me find x please

Answers

Step-by-step explanation:

x² + (2x)² = (√180)²

x² + 4x² = 180

5x² = 180

x² = 180/5

x² = 36

x = √36 (and no - √36 because x ≥ 0)

x = 6

Quick algebra 1 assignment for some points!
I LOST MY OLD ACCOUNT I WAS A GENIUS, :((( SO NOW I GOTTA MAKE A NEW ONE !!! :((

Only answer if you know the answer, quick shout-out to Yeony2202, tysm for the help!


Oh by the way this is just a section of the real assignment, the assignment calls for you to make an app that people can play to learn inverse variation & direct variation and stuff.


Hope that helps solve this! :)

Quick algebra 1 assignment for some points!I LOST MY OLD ACCOUNT I WAS A GENIUS, :((( SO NOW I GOTTA

Answers

Inverse variation is the relationship that occurs between two quantities in which one decreases as the other increases.

Inverse variation

The types of variation which exists are;

Inverse variationDirect variationJoint variation

For instance;

The relationship between John's speed and distance is inversely proportional. If John's speed is 5km/h, his distance is 6km. Find John's distance when his speed increases to 6km/h.

s = k / d

where,

k = constant of proportionality

s = k / d

5 = k/6

30 = k

Find d when s = 6km/h

s = k / d

6 = 30/d

6d = 30

d = 30/6

d = 5 km

Learn more about inverse variation:

https://brainly.com/question/10929089

#SPJ1

Other Questions
Answer for bonus points!! How does genetic engineering make the production of a human hormone safer, cheaper, and faster? Is genetic engineering always beneficial? Explain. evaluate the expression and enter expression and enter your answer in the box below -3(6+2)+2328 What is public policy in government?. If the correlation between the growth of a stock and the growth of the economy as a whole is close to 1, then this would be a good stock to hold during a recession when the economy shrinks.Group of answer choicesTrueFalse You want to be able to withdraw $30,000 from your account each year for 25 years after you retire.You expect to retire in 15 years.If your account earns 9% interest, how much will you need to deposit each year until retirement to achieve your retirement goals? How do you get rid of the red part in the corner of your eye? what is the purpose of them and what do they do? The parking garage has 9 rows with 10 parking spaces in each row. There are 8 empty spaces.How many spaces are filled? a tender, proposal, or quotation submitted in response to a solicitation (i.e., rfp, rfq) from a contracting authority is known as what? a tender, proposal, or quotation submitted in response to a solicitation (i.e., rfp, rfq) from a contracting authority is known as what? bond broker bid reverse auction Use the given conditions to write an equation for the line in point-slope form and general form Passing through (4.-5) and perpendicular to the line whose equation is x - 5y - 7=0 Which expression means the same as a number minus 5? difference between qualified and ordinary dividends if a cone has a volume of 28.63 and a diameter of 4.5 what is the height the sum of 3 and a number Which of the following is a problem created by the construction of the Aswan High Dam?A. regular water supply C. decreased occurrence of droughtB. increased irrigation D. decreased silt deposits on the surrounding farmland Reread the poem "Delotavo's Diaspora". Then, cite pieces of evidence from the poem to support the general statement given below."Leaving the country to work abroad is a difficult decision for a Filipino family." the process observed when an economy's production possibility frontier is shifted outward is: a. Explain what is meant by the time inconsistency problem of monetary policy. Why does it arise? Can it be solved? (5 points) b. According to theory discussed in the course, we should treat currencies as financial assets in the short run. How is the nominal exchange rate determined according to this theory? Explain which factors affect the decision of an investor considering investing in a given currency. (5 points) c. Consider the search and matching model discussed in the course. The Beveridge curve, the job-creation condition and the wage setting curves, respectively, are given by: u= + 0q (0) (r+)pc p-w- = 0, q(0) w = (1-B)z + Bp(1 + c0). Analyse the effects of an increase in z. (5 points) You need to accumulate $89,263 for your son's education. You have decided to place equal year-end deposits in a savings account for the next 4 years. The savings account pays 10.45 percent per year, compounded annually. How much will each annual payment be?Round the answer to two decimal places. 60 yo M presents with dull epigastric pain that radiated to the back, accompanied by weight loss, dark uring, and clay colered stool he is a heavy drinker and smoker. What is the most likely diagnosis?